Probability proof by combinatorial argument

Click For Summary
The discussion centers on proving a combinatorial identity involving the selection of items from two subsets. The left side of the equation represents the total ways to choose r items from a combined set of m and n items. The right side breaks down the selection process by considering different combinations of items taken from each subset. Participants emphasize the need to establish a correspondence between choices on both sides of the equation, reinforcing that selecting r items involves choosing a specific number from each subset. The conversation highlights the importance of structuring the proof to clearly demonstrate this relationship.
Proggy99
Messages
49
Reaction score
0

Homework Statement


By a combinatorial argument, prove that for r \leq n and r \leq m,
(^{n+m}_{r}) = (^{m}_{0})(^{n}_{r}) + (^{m}_{1})(^{n}_{r-1}) + ... + (^{m}_{r})(^{n}_{0})


Homework Equations





The Attempt at a Solution


I need some direction on how to start this problem. It is the only homework problem I am not sure of how to approach it.
 
Physics news on Phys.org
okay, so here is my attempt at beginning the solution

The left side gives the number of ways that you can pick r total items from a set made up of two subsets of items with m and n items in each subset.

The right side gives a series of permutations including how to pick no items from the first subset and all r items from the second, then how to pick 1 item from the first subset and the rest from the second, then 2 items from the first subset and the rest from the second, and continuing on until you are choosing all r items from the first subset and no items from the second.

I am just not sure how to go about putting this in proof form
 
So a choice on the right side must correspond to one of the choices on the left side and vice versa, right? I think that's exactly what you want to say, and I would call it a 'proof'.
 
You can argue that in order to pick r items from m + n items, you have to pick x from the m items and r - x from the n items.
 
Question: A clock's minute hand has length 4 and its hour hand has length 3. What is the distance between the tips at the moment when it is increasing most rapidly?(Putnam Exam Question) Answer: Making assumption that both the hands moves at constant angular velocities, the answer is ## \sqrt{7} .## But don't you think this assumption is somewhat doubtful and wrong?

Similar threads

  • · Replies 3 ·
Replies
3
Views
1K
Replies
7
Views
2K
  • · Replies 1 ·
Replies
1
Views
2K
  • · Replies 6 ·
Replies
6
Views
2K
  • · Replies 1 ·
Replies
1
Views
1K
  • · Replies 7 ·
Replies
7
Views
1K
Replies
3
Views
2K
  • · Replies 7 ·
Replies
7
Views
2K
  • · Replies 3 ·
Replies
3
Views
2K
  • · Replies 4 ·
Replies
4
Views
2K